LSAT and Law School Admissions Forum

Get expert LSAT preparation and law school admissions advice from PowerScore Test Preparation.

User avatar
 Dave Killoran
PowerScore Staff
  • PowerScore Staff
  • Posts: 5853
  • Joined: Mar 25, 2011
|
#59725
Complete Question Explanation
(The complete setup for this game can be found here: lsat/viewtopic.php?t=6078)

The correct answer choice is (E)

The question stem creates a KO block. Since no obvious diagramming inferences can be made, reuse information, as was done in question #5.

The hypothetical from Question #1 applies since K and O are next to each other (remember, the first and last variables are next to each other); and this hypothetical proves that L can sit directly between K and M. That is sufficient to eliminate answer choice (A). The work from question #2 also applies and is sufficient to eliminate answer choice (D). Question #3 does not apply. Question #4 does apply and proves that L can sit between M and O (not in the answers) or M and K (already known from question #1). The hypothetical from question #6 must be examined closely. In spite of the fact that M and K rotate in the dual-option, K will always sit next to O, and thus the hypothetical reveals that L can sit between K and N as well as M and N. This information eliminates answer choices (B) and (C). Answer choice (E) is therefore correct via process of elimination.

This question again reveals the power of reusing previous work. With minimal effort all four incorrect answer choices are eliminated. And since the hypotheticals can be visually scanned at a high rate, the question takes less time than a question requiring a new diagram.

Circular Linearity games appear once in a blue moon, and it is not likely that one will appear on your test. Nevertheless, it is worthwhile to be acquainted with the basic principles in case such a game happens to appear.
 bk1111
  • Posts: 103
  • Joined: Apr 22, 2017
|
#36913
Hello,

What is the best way to approach #7? I used my global set-up, but I realized it would require too many scenarios to get to the right answer and thus becomes very time consuming. Is there a better approach to this type of question in general? Please help. Thank you.
 Luke Haqq
PowerScore Staff
  • PowerScore Staff
  • Posts: 747
  • Joined: Apr 26, 2012
|
#37434
Hi bk1111,

I think the best way to approach this one is move forward with two alternate diagrams that have K and O together. This won't look quite like your diagram since yours will be circular, the underlying spaces 1 through 6 will still be the same. In one of the alternate diagrams, put O first, and in the other, put K first:

O, K, 3?, 4 ?, 5?, 6?

K, O, 3?, 4 ?, 5?, 6?

As far as I can see, this question requires you to try each answer--with only one proving that it must be true (specifically, it must be true that something cannot happen). While it might seem tedious, for each possible answer choice, you could quickly plug in the variables they give you into the above two alternate possibilities. That would result in showing that (A) through (D) could be true.

However, consider plugging in what answer (E) supposes--that L sits between N and O (i.e., O-L-N or N-L-O). In the above diagrams (remember that 1 and 6 wrap around because it should be circular), this would look like:

O, K, 3?, 4?, L, N
  • (1) this can be rewritten as: O, K, P/M, P/M, L, N
    (2) we know K and M can't be together, so we can rewrite: O, K, P, M, L, N
    (3) however, this violates the requirement that P and N must be together
    (4) therefore, this possible combination starting with O can't work
K, O, L, N, 5?, 6?
  • (1) this can be rewritten as: K, O, L, N, P/M, P/M
    (2) we can rewrite and ensure P and N are together, but this puts M in spot 6--this would violate the rule that K and M cannot be together
    (3) we can rewrite to make sure M and K are not together by putting M is spot 5, but then P and N are not together
    (4) therefore, this possible alternative starting with K can't work
In other words, using this alternative-possibilities approach with answer (E), you'll see that neither way the possibilities would be diagrammed out would work with the game's rules.

Hope that helps!
 AJH
  • Posts: 15
  • Joined: Nov 20, 2017
|
#43052
I also have a question regarding #7. I tried the approach of testing each answer, but chose answer B. My diagram was KOMPNL for answer B. I assumed this was the correct answer because it places M away from both/either L &N. I see that answer choice E is also impossible, but I am wondering what I am missing with answer B that is incorrect.
 Emily Haney-Caron
PowerScore Staff
  • PowerScore Staff
  • Posts: 577
  • Joined: Jan 12, 2012
|
#43338
Hi AJH,

M doesn't have to be next to L or N; M can be anywhere, but L has to be next to either M or N (or both). In your solution, since L is still next to N, you don't violate that rule.

Hope that helps!
 deck1134
  • Posts: 160
  • Joined: Jun 11, 2018
|
#49805
I watched Jon's module but struggled a lot with number 7. Is there a way to do this question? The usual place and go method from the video works well on 1-6, but this question foils that.

I drew out the spokes, as requested, but couldn't find a good way to say that (E) is the answer. Any thoughts? I brute forced the problem, but I screwed up one rule and missed the question. Any help would be much appreciated.
 JKP2018
  • Posts: 9
  • Joined: Sep 01, 2018
|
#57945
I would also like clarity on this question. The answer in the Bible talks about reusing previous work, but I didn't include KO in many of my diagrams; in most answer they weren't required, and I didn't place them in my diagram. That saved me time up until this question, when I was left with nothing but brute force diagraming every choice ... and then the right answer turned out to be e, the last one, and I spent three minutes on this question alone.

Is there nothing else besides brute force and look at past work for such questions?
 Rachael Wilkenfeld
PowerScore Staff
  • PowerScore Staff
  • Posts: 1358
  • Joined: Dec 15, 2011
|
#57977
Hi Deck and JKP,

Since you both have pretty much the same question---how do you deal with number 7?

One good strategy for a question like this one, where you don't have past work to help you is to think about restricted variables. As a cannot be true question, you are searching for a rule violation. That's why it's best to start with the variables with a lot of restrictions. What variables in this game are generally hard to place? What would implicate a lot of rules?

The NP rule is still very strong in this game, so I'm automatically looking askew at answer choices with N or P linked with either O or K. I wouldn't rule out the other answer choices yet, but I would start with those that triggered the NP rule.

That gives us answer choice (B) and answer choice (E) as starting possibilities.

Your diagram should be drawn in a circle, but because I'm using a text box, I'll need you to use your imagination to turn my line of variables into a circle.

For answer choice (B), it asks if L could sit between K and N. Let's imagine we try it out. What would that look like?
We could put OKLNPM. That doesn't violate any of our rules. So answer choice (B) is eliminated.

Let's look at answer choice (E). Can we put L between N and O? We could try KOLNPM, but that would put M and K next to one another, since it's a circle, not a line. We could also reverse that (MPNLOK), but we have the same error. So answer choice (E) is our correct answer for the cannot be true question.

Sometimes, there's no faster way than plug and go. But you can gain speed and time by being smart about which answer choices you try first.

Hope that helps!

Get the most out of your LSAT Prep Plus subscription.

Analyze and track your performance with our Testing and Analytics Package.